Search found 51 matches


the reasoning above makes sense, but I'm still confused between A and C. What if Kreemo purchases more beans, but as the machines which it uses for blending are same as the other manufacturers' , its coffee will be no different from the other blends. Basically, the amount of Colombian thing in Kree...

by lkm

Sun Apr 25, 2010 5:06 pm
Forum: Critical Reasoning
Topic: CR
Replies: 6
Views: 2441

reply2spg wrote:what is wrong with B here?
Option B is the direct paraphrase of premise. So, it is a fact not the assumption. And you have to find the assumption here to lead to the conclusion.

by lkm

Wed Apr 21, 2010 9:37 am
Forum: Critical Reasoning
Topic: West Side School, LSAT CR
Replies: 6
Views: 2934

Hi Stephen has already explained it in detail. However, here is my explanation in shorter form. Statement A: It states that all student with disabilities have special educational needs. However, it leaves an open space here for the other students who might have special educational needs BUT they don...

by lkm

Mon Apr 19, 2010 6:56 pm
Forum: Critical Reasoning
Topic: West Side School, LSAT CR
Replies: 6
Views: 2934

Based on what lkm said: "If people choose to walk over ride, then there will be less vehicles on the road and it leads to less congestion or may be no congestion." By extending it little bit more you get the below cause -effect. Less vehicles means less congestion and less congestion mean...

by lkm

Mon Apr 19, 2010 12:10 am
Forum: Critical Reasoning
Topic: lsat strenghtening and assumption
Replies: 9
Views: 2491

Shawshank wrote:
I dont agree with this explanation.. Still have a doubt as to why B... any other takers who can explain better..
Phoenix.. u sure,,, OA -- B
Any reason for disagreement. I would like to discuss.

Thanks

by lkm

Thu Apr 15, 2010 10:54 am
Forum: Critical Reasoning
Topic: lsat strenghtening and assumption
Replies: 9
Views: 2491

Hi Kevin,

How to solve question number 27?

Please reply.

Thanks,

by lkm

Wed Apr 14, 2010 12:29 pm
Forum: Reading Comprehension
Topic: interesting RC passage
Replies: 6
Views: 1896

As per the set the OA for second one is B any views !!! Well none of the answer choices are even near to strengthen the argument. However, here is my stake. Which one of the following, if true, most strengthens the arguement? A) If automobile passengers who never drive walk instead of ride, there w...

by lkm

Tue Apr 13, 2010 1:16 pm
Forum: Critical Reasoning
Topic: lsat strenghtening and assumption
Replies: 9
Views: 2491
by lkm

Tue Apr 13, 2010 12:33 pm
Forum: Sentence Correction
Topic: Disabling effect - Conditionals
Replies: 22
Views: 4389

If 60! is written out as an integer, with how many consecutive 0’s will that integer end? 1]6 2]12 3]14 4]42 5]56 What is a quick way to solve this? The quickest way to solve this is to identify the number of 5's in the 60! So, five will appear in the following numbers:- 5 10 15 20 25 (5x5) --> T...

by lkm

Mon Apr 12, 2010 8:48 pm
Forum: Problem Solving
Topic: number properties
Replies: 3
Views: 1164
by lkm

Sat Apr 10, 2010 2:12 pm
Forum: Sentence Correction
Topic: water molecules
Replies: 26
Views: 4940

As an ADJECTIVE: In the meetings, I often bring a chocolate to quench my hunger during the breaks. Here TO QUENCH is used as an ADJECTIVE to modify a chocolate . REMEMBER: Adjective can modify only adjectives or noun. So, 100% you can place NOUN (in the above sentence, ME is pronoun used after to q...

by lkm

Sat Apr 10, 2010 1:59 pm
Forum: Sentence Correction
Topic: usage of infinitives
Replies: 5
Views: 1244

Please underline the sentence.

IMO answer should be (B) because in the sentence, THAT will refer back to immediate preceding noun, which FACTORS.

Since, FACTORS is plural. Hence, KEEP should comes instead of KEEPS.

Hence, go with (B).

by lkm

Thu Apr 08, 2010 12:50 pm
Forum: Sentence Correction
Topic: Interest rate
Replies: 8
Views: 1881

I came across a problem, which had an explantion that confused me, when I was doing my OG 12. It said that infinitives cannot follow a noun. I managed to get a few counter examples, which I am posting below as well. Please provide me an explanation on this. eg: I asked a diffiuclt question to snub ...

by lkm

Thu Apr 08, 2010 12:31 pm
Forum: Sentence Correction
Topic: usage of infinitives
Replies: 5
Views: 1244

Can you please explain what will be answer if we are asked "find out how many 20 are there in 30!" ? I mean in this case also we have to calculate number of 5s in 30 ! . Thanks. Dude! Here is the simple logic for finding all the numbers:- Let's say number to find = N Then factorize the nu...

by lkm

Wed Apr 07, 2010 9:12 pm
Forum: Problem Solving
Topic: How many 10s we can get out of 30! ?
Replies: 9
Views: 1627

To relieve anxiety, moderate exercise can be equally effective as, and less addictive than, most sedatives. a. effective as, and b. as effective as, while being c. effectively equal to, but d. as effective as, and e. effective, and IMO (D) A) Correct IDIOM is either "EQUAL TO..." or "...

by lkm

Tue Apr 06, 2010 12:12 pm
Forum: Sentence Correction
Topic: confused
Replies: 7
Views: 1677